0 Daumen
197 Aufrufe

Aufgabe:

Ist ein Widerlegungsbeweis der Collatz-Vermutung akzeptabel, insbesondere dieser?


Problem/Ansatz:

Hallo liebe Liebhaber der Zahlentheorie und insbesondere der Collatz-Vermutung. Ich möchte Ihnen einen kurzen und ungewöhnlichen Widerlegungsbeweis der Collatz-Vermutung vorstellen, der auf einem Widerspruch basiert. Aber ist dieser Widerlegungsbeweis auch dann noch akzeptabel, wenn die Collatz-Vermutung für alle Zahlen bis 2^68 (ca. 2,95 x 10^20) rechnerisch bestätigt wurde? Ich teile den Link mit Ihnen und freue mich auf Ihre Anmerkungen: https://www.researchgate.net/publication/391596890_A_mathematical_letter_that_disproves_by_contradiction_the_Collatz_conjecture

Avatar vor von
Aber ist dieser Widerlegungsbeweis auch dann noch akzeptabel, wenn die Collatz-Vermutung für alle Zahlen bis 268 (ca. 2,95 x 1020) rechnerisch bestätigt wurde?

Ohne deine "Widerlegung" gelesen zu haben: Wer die gerade zitierte Frage stellt, hat von Logik null Ahnung. Dir fehlen jegliche Grundlagen.

Du kennst mich nicht einmal und beleidigst mich nur aus Rassismus. Lesen Sie meinen Beweis, um zu sehen, dass Sie derjenige sind, der keine mathematischen Prinzipien kennt.

Er beleidigt nicht. Und Rassismus ist das auch nicht.

Wenn eine Aussage bereits für \(2^{68}\) Werte bestätigt wurde, ist das zwar noch kein endgültiger Beweis, allerdings darf die Korrektheit deines Beweises durchaus angezweifelt werden, da er mit sehr großer Wahrscheinlichkeit ohnehin falsch ist.

In der zitierten Frage liegt kein Logik-Problem vor. Wohl aber darin hier irgendwo Rassismus zu entdecken. Und ob ein Beweis wahrscheinlich falsch ist oder wahrscheinlich richtig, ist irrelevant. Die Richtigkeit eines Beweises hängt an streng formalen Kriterien, und nicht an Wahrscheinlichkeiten aufgrund einer Anzahl (wie hoch sie auch sei) Beispielen oder Gegenbeispielen.

Er stellt doch durch die zitierte Frage seinen eigenen Beweis ohnehin in Frage. Wenn er eine ernsthafte Antwort möchte, sollte er sich doch viel lieber an die entsprechenden mathematischen Journale wenden und nicht an ein Matheforum.

Ich hab mich mit der CV nicht beschäftigt, bin davon ausgegangen, dass sie beginnt mit "für alle....". Wenn AL zeigen kann, dass es eine Zahl gibt, die die CV widerlegt, ist das kein Widerspruch dazu, dass sie für endlich viele Zahlen bestätigt ist. Aber wenn Ihr die Frage an sich meint (ob der Beweis akzeptabel sei), ist das in der Tat etwas schräg. Dann wäre zu klären, was er mit "akzeptabel" meint.

Deinem zweiten Satz stimme ich uneingeschränkt zu. Das Niveau dieses Matheforums ist dafür (und für vieles andere) ungeeignet.

Ich habe die Meldung gelöscht, dass diese Frage ein Duplikat ist.

Es geht zwar in beiden Beiträgen um die Collatz-Vermutung, aber ein Duplikat sehe ich nicht.

https://www.mathelounge.de/1060005/finally-collatz-conjecture-possible-expressions-numbers

Dieses Forum ist wahrscheinlich nicht der beste Ort für so spezifische Fragen. Foren wie MathOverflow wären vermutlich besser geeignet, da dort mehrere Personen mit der Thematik vertraut sind.

Vielen Dank.

Der Link meiner Frage auf Mathoverflow:

https://mathoverflow.net/questions/492490/a-strange-disproof-of-the-collatz-conjecture

Mit freundlichen Grüssen.

Ein anderes Problem?

Stell deine Frage

Willkommen bei der Mathelounge! Stell deine Frage einfach und kostenlos

x
Made by a lovely community